Construcción explícita de (12,12)(12,12)(\frac{1}{2}, \frac{1}{2}) representación del grupo de Lorentz

Para la representación vectorial del grupo de Lorentz (en realidad el álgebra), el j 1 generador es

(1) j 1 = i ( 0 0 0 0 0 0 0 0 0 0 0 1 0 0 1 0 ) ,

mientras que lo que obtengo aplicando tratando de resolverlo desde el ( 1 2 , 1 2 ) la representacion es

(2) j 1 = ( 0 1 / 2 1 / 2 0 1 / 2 0 0 1 / 2 1 / 2 0 0 1 / 2 0 1 / 2 1 / 2 0 )

Obviamente hay un error en mi cálculo, la pregunta es dónde y cómo rectificarlo. Así es como llegué al cálculo anterior.

Construir las representaciones irreductibles del grupo homogéneo de Lorentz, a partir del j m v generadores, se define j = { j 1 , j 2 , j 3 } como j 1 = j 23 y así sucesivamente por permutaciones cíclicas y k = { k 1 , k 2 , k 3 } como k i = j 0 i . Luego, definiendo

(3) A = 1 2 ( j + i k ) , B = 1 2 ( j i k )

uno obtiene dos independientes S tu ( 2 ) álgebras, ya que

(4) [ A i , A j ] = i ϵ i j k A k ,

(5) [ B i , B j ] = i ϵ i j k B k ,

(6) [ A i , B j ] = 0.

De este modo A se puede representar mediante las matrices habituales de momento angular j a , a ( A ) , dónde A es el valor máximo (entero o medio entero) y a , a = A , . . . , + A . Del mismo modo para B . Combinando los dos para el grupo de Lorentz, en términos de índices tenemos

(7) ( A ) a b , a b = j a a ( A ) d b b

(8) ( B ) a b , a b = d a a j b b ( B )

Ahora aquí comienza mi problema. Según tengo entendido, en forma matricial estos dos son

(9) A = j ( A ) 1 2 b + 1

y

(10) B = 1 2 a + 1 j ( B ) .

1 representa, por supuesto, la matriz unitaria de las dimensiones apropiadas y es el producto directo de la matriz.

Usando estas ecuaciones para A = 1 2 = B , el j son 1 2 σ i , la mitad de las matrices de Pauli y lo que obtengo es

(11) A 1 = 1 2 σ 1 1 2 = ( 0 1 / 2 1 / 2 0 ) ( 1 0 0 1 ) =

(12) = ( 0 ( 1 0 0 1 ) 1 2 ( 1 0 0 1 ) 1 2 ( 1 0 0 1 ) 0 ( 1 0 0 1 ) ) = ( 0 0 1 / 2 0 0 0 0 1 / 2 1 / 2 0 0 0 0 1 / 2 0 0 )

y de manera similar

(13) B 1 = 1 2 1 2 σ 1 = ( 1 0 0 1 ) ( 0 1 / 2 1 / 2 0 ) = ( 0 1 / 2 0 0 1 / 2 0 0 0 0 0 0 1 / 2 0 0 1 / 2 0 )

De las definiciones de A y B , ecuación (3), j = A + B , por lo tanto j 1 = A 1 + B 1 y

(14) j 1 = ( 0 1 / 2 1 / 2 0 1 / 2 0 0 1 / 2 1 / 2 0 0 1 / 2 0 1 / 2 1 / 2 0 )

que apenas se parece a la j 1 para la representación vectorial.

En algún lugar a lo largo de la línea hay un error, pero no lo veo. Cualquier ayuda y consejo será muy apreciado.

Consulta mi respuesta aquí physics.stackexchange.com/q/321292
Creo que hay dos problemas: 1. no complejiste conjugado B 2. no estás mirando estas matrices en la base correcta

Respuestas (1)

Tú, de hecho, no cometiste ningún error. Simplemente no siguió el problema hasta el final, cambiando las bases para reducir su representación reducible. De hecho, independientemente de su uso de incrustación de Lorentz, esta es solo la composición del producto de Kronecker de dos representaciones de giro 1/2 en la suma de Kronecker de una representación de giro 1 y una representación de giro 0 ... suma de grado de momento angular. ¡En realidad nunca usaste ningún potenciador!

La representación reducible del momento angular que encontraste, (14),

Δ ( j 1 ) = 1 2 ( 0 1 1 0 1 0 0 1 1 0 0 1 0 1 1 0 ) ,
Δ ( j 3 ) = diag (1,0,0,-1), etc., se puede reducir a la suma directa de un triplete (giro uno) y un singlete (giro 0, ¡así que los generadores se realizan trivialmente por 0!) rep, por el Clebsch ortogonal –Matriz de Gordon,
C = ( 1 0 0 0 0 1 / 2 1 / 2 0 0 1 / 2 1 / 2 0 0 0 0 1 ) .
Es decir,
C 1 Δ ( j 1 ) C = 1 2 ( 0 1 0 0 1 0 0 1 0 0 0 0 0 1 0 0 ) .
El tercer componente de los 4 vectores se ha desacoplado (¡es el singlete!), mientras que los otros 3 componentes constituyen solo
j 1 = 1 2 ( 0 1 0 1 0 1 0 1 0 ) ,
el generador de tripletes en la base esférica . La nota al pie en ese artículo, o esta respuesta , le brinda la transformación de similitud a la expresión de base cartesiana que escribió, (1), después de que la fila 0 y las columnas también se eliminen. Por supuesto, Δ ( j 3 ) = diag (1,0,0,-1) no se vio afectado por el Clebsching anterior para producir solo j 3 = diag(1,0,-1).

Puede convencerse de la reducción análoga con la misma matriz de Clebsch-Gordan para Δ ( j 2 ) . A continuación, puede ajustar este subespacio de representación en un bloque de 3 por 3 (el inferior derecho) de la representación de 4 vectores que está utilizando en 1. Como última verificación para apreciar la estructura, calcule el invariante cuadrático completo de Casimir. y su reducción a una identidad 3d y una entrada cero para el singlete.

De hecho, parece que me perdí la transformación de similitud. Más concretamente, tengo que volver a la teoría de las representaciones tensoriales y cómo se descomponen en diferentes representaciones irreducibles. Toda ayuda y consejo es muy apreciada.
Tal vez el problema 4 aquí podría ayudar con el idioma.